Difference between revisions of "Aufgaben:Exercise 5.2: Determination of the Frequency Response"

From LNTwww
 
(22 intermediate revisions by 4 users not shown)
Line 1: Line 1:
  
{{quiz-Header|Buchseite=Stochastische Signaltheorie/Stochastische Systemtheorie
+
{{quiz-Header|Buchseite=Theory_of_Stochastic_Signals/Stochastic_System_Theory
 
}}
 
}}
  
[[File:P_ID492__Sto_A_5_2.png|right|]]
+
[[File:EN_Sto_A_5_2_neu2.png|right|frame|Measuring the frequency response  $H(f)$]]
:Wir betrachten die abgebildete Messanordnung zur Bestimmung des blau hervorgehobenen Frequenzgangs <i>H</i>(<i>f</i>). Das Eingangssignal <i>x</i>(<i>t</i>) ist weißes Gaußsches Rauschen mit der Rauschleistungsdichte <i>N</i><sub>0</sub> = 10<sup>&ndash;10</sup> W/Hz. Somit gilt für die AKF:
+
We consider the illustrated measurement setup for the determination of the frequency response&nbsp; $H(f)$&nbsp; highlighted in blue.  
:$$\varphi _x ( \tau ) = \frac{N_0 }{2} \cdot \delta ( \tau  ).$$
+
*The input signal&nbsp; $x(t)$&nbsp; is white Gaussian noise with the noise power density&nbsp; $N_0 = 10^{-10} \hspace{0.05cm} \rm W/Hz$.  
 +
*Thus,&nbsp; the auto-correlation function&nbsp; $\rm (ACF)$&nbsp; is:
 +
:$$\varphi _x ( \tau ) = {N_0 }/{2} \cdot \delta ( \tau  ).$$
 +
*The measured cross-correlation function&nbsp; $\rm (CCF)$&nbsp; between the signals&nbsp; $x(t)$&nbsp; and&nbsp; $y(t)$&nbsp; can be approximated as follows <br>$($valid only for positive times&nbsp; $t)$:
 +
:$$\varphi _{xy} \left( \tau  \right) = K \cdot {\rm{e}}^{ - \tau /T_0 },\hspace{0.5cm}\text{with } \ K = 0.628 \cdot 10^{-12} \hspace{0.05cm} {\rm W}, \  T_0 = 1 \hspace{0.05cm} \rm ms.$$
 +
*The ACF&nbsp; $\varphi_y(\tau)$&nbsp; of the output signal&nbsp; $y(t)$ is also measured.
  
:Die gemessene Kreuzkorrelationsfunktion (KKF) zwischen den Signalen <i>x</i>(<i>t</i>) und <i>y</i>(<i>t</i>) kann mit <i>K</i> = 0.628 &middot; 10<sup>&ndash;12</sup> W und <i>T</i><sub>0</sub> = 1 ms wie folgt angenähert werden (nur gültig für positive Zeiten):
 
:$$\varphi _{xy} \left( \tau  \right) = K \cdot {\rm{e}}^{ - \tau /T_0 } .$$
 
  
:Gemessen wird außerdem die AKF <i>&phi;<sub>y</sub></i>(<i>&tau;</i>) des Ausgangssignals.
 
  
  
''Hinweise:''
+
Notes:  
*Die Aufgabe gehört zum  Kapitel [[Stochastische_Signaltheorie/Stochastische_Systemtheorie|Stochastische Systemtheorie]].
+
*The exercise belongs to the chapter&nbsp; [[Theory_of_Stochastic_Signals/Stochastic_System_Theory|Stochastic System Theory]].
*Bezug genommen wird auch auf die Kapitel [[Stochastische_Signaltheorie/Gaußverteilte_Zufallsgrößen|Gaußverteilte Zufallsgrößen]] sowie [[Stochastische_Signaltheorie/Leistungsdichtespektrum_(LDS)|Leistungsdichtespektrum]].  
+
*Reference is also made to the chapter&nbsp; [[Theory_of_Stochastic_Signals/Power-Spectral_Density|Power-Spectral Density]].  
*Sollte die Eingabe des Zahlenwertes &bdquo;0&rdquo; erforderlich sein, so geben Sie bitte &bdquo;0.&rdquo; ein.
+
*Please also note the following Fourier transform&nbsp; $($in&nbsp; $\omega)$:
*Benutzen Sie, falls nötig, die nachfolgenden Gleichungen:
+
:$$H( \omega  ) = \frac{1}{{1 + {\rm{j}}\cdot \omega /\omega _0 }}\hspace{0.15cm}\bullet\!\!\!-\!\!\!-\!\!\!-\!\!\circ\,\hspace{0.15cm}h(t) = \omega _0  \cdot {\rm{e}}^{ - \omega _0 t} \hspace{0.3cm}(t \ge 0).$$
:$${\rm Q}(x) \approx \frac{1}{{\sqrt {2{\rm{\pi }}\cdot x}} \cdot {\rm{e}}^{ - x^2 /2} \quad {\rm{(f\ddot{u}r }}\;{\rm{grösse }}\;x{\rm{)}}{\rm{,}}$$
+
:For negative&nbsp; $t$&ndash;values,&nbsp; on the other hand,&nbsp;  $h(t) =0$&nbsp; at all times.
:$$\int {\rm{cos}}^{\rm{2}}( {ax} )\hspace{0.1cm}{\rm{d}}x = \frac{1}{2} \cdot x + \frac{1}{4a} \cdot \sin ( {2ax} ),$$
 
:$$\int {\cos ^4 } ( {ax} )\hspace{0.1cm}{\rm{d}}x = \frac{3}{8} \cdot x + \frac{1}{4a} \cdot \sin ( {2ax} ) + \frac{1}{32a} \cdot \sin ( {4ax} ).$$
 
  
:<b>Hinweis:</b> Die Aufgabe bezieht sich auf die theoretischen Grundlagen von Kapitel 4.6 und Kapitel 5.1. Beachten Sie bitte auch die folgende Fouriertransformation (in <i>&omega;</i>):
 
:$$H( \omega  ) = \frac{1}{{1 + {\rm{j}}\cdot \omega /\omega _0 }}\bullet\!\!\!-\!\!\!-\!\!\!-\!\!\circ\,h(t) = \omega _0  \cdot {\rm{e}}^{ - \omega _0 t} \hspace{0.3cm}(t \ge 0).$$
 
:Für negative <i>t</i>-Werte ist dagegen <i>h</i>(<i>t</i>) stets 0.
 
  
===Fragebogen===
+
 
 +
===Questions===
  
 
<quiz display=simple>
 
<quiz display=simple>
{Welche der folgenden Aussagen treffen zu? Man kann den Frequenzgang <i>H</i>(<i>f</i>) nach Betrag und Phase vollständig bestimmen, wenn:
+
{Which of the following statements is true?&nbsp; One can completely determine the frequency response&nbsp; $H(f)$&nbsp; by magnitude and phase if
 
|type="[]"}
 
|type="[]"}
- die Funktionen <i>&phi;<sub>x</sub></i>(<i>&tau;</i>) und <i>&phi;<sub>y</sub></i>(<i>&tau;</i>) bekannt sind,
+
- the functions&nbsp; $\varphi_x(\tau)$&nbsp; and&nbsp; $\varphi_y(\tau)$&nbsp; are known,
+ die Funktionen <i>&phi;<sub>x</sub></i>(<i>&tau;</i>) und <i>&phi;<sub>xy</sub></i>(<i>&tau;</i>) bekannt sind,
+
+ the functions&nbsp; $\varphi_x(\tau)$&nbsp; and&nbsp; $\varphi_{xy}(\tau)$&nbsp; are known,
+ die Funktionen <i>&phi;<sub>xy</sub></i>(<i>&tau;</i>) und <i>&phi;<sub>y</sub></i>(<i>&tau;</i>) bekannt sind.
+
+ the functions&nbsp; $\varphi_{xy}(\tau)$&nbsp; and&nbsp; $\varphi_y(\tau)$&nbsp; are known.
  
  
{Berechnen Sie die Impulsantwort <i>h</i>(<i>t</i>). Welcher Wert ergibt sich für <i>t</i> = <i>T</i><sub>0</sub>?
+
{Calculate the impulse response&nbsp; $h(t)$.&nbsp; What is the value for&nbsp; $t=T_0$?
 
|type="{}"}
 
|type="{}"}
$h(t = T_0)$ = { 4.62 3% } $\cdot 10^{-3} 1/s$
+
$h(t = T_0) \ = \ $ { 4.62 3% } $\ \cdot 10^{-3} \ \rm 1/s$
  
  
{Wie lautet der Frequenzgang <i>H</i>(<i>f</i>)? Welcher Wert ergibt sich für <i>f</i> = 0?
+
{What is the frequency response&nbsp; $H(f)$?&nbsp; What value results for $f= 0$?
 
|type="{}"}
 
|type="{}"}
$H(f = 0)$ = { 0.5 3% }
+
$H(f = 0) \ =  \ $ { 0.5 3% }
  
  
{Berechnen Sie das Leistungsdichtespektrum des Ausgangssignals <i>y</i>(<i>t</i>). Welcher Wert ergibt sich bei <i>f</i> = 1/(2&pi;<i>T</i><sub>0</sub>)?
+
{Calculate the power-spectral density of the output signal&nbsp; $y(t)$.&nbsp; What value results for frequency&nbsp; $f = 1/(2\pi T_0)$?
 
|type="{}"}
 
|type="{}"}
$\phi_y(f = 1/(2\pi T_0))$ = { 6.25 3% } $\cdot 10^{-12}\ W/Hz$
+
${\it \Phi}_y(f = 1/(2\pi T_0)) \ =  \ $ { 6.25 3% } $\ \cdot 10^{-12}\ \rm W/Hz$
  
  
Line 54: Line 52:
 
</quiz>
 
</quiz>
  
===Musterlösung===
+
===Solution===
 
{{ML-Kopf}}
 
{{ML-Kopf}}
:<b>1.</b>&nbsp;&nbsp;Bei der AKF-Berechnung gehen Phasenbeziehungen verloren. Die zugehörigen Funktionen <i>&Phi;<sub>x</sub></i>(<i>f</i>) und <i>&Phi;</i><i><sub>y</sub></i>(<i>f</i>) im Spektralbereich sind rein reell, so dass nur der Betrag |<i>H</i>(<i>f</i>)| angegeben werden kann.
+
'''(1)'''&nbsp; <u>Statements 2 and 3</u> are true.
 
+
*The following equations are valid:
:Die <u>Aussagen 2 und 3</u> sind zutreffend, da folgende Gleichungen gelten:
 
 
:$$\varphi _{xy} ( \tau ) = h( \tau ) * \varphi _x ( \tau )\quad  \Rightarrow \quad H( f ) = \frac{{{\it \Phi} _{xy} ( f )}}{{{\it \Phi} _x ( f )}},$$
 
:$$\varphi _{xy} ( \tau ) = h( \tau ) * \varphi _x ( \tau )\quad  \Rightarrow \quad H( f ) = \frac{{{\it \Phi} _{xy} ( f )}}{{{\it \Phi} _x ( f )}},$$
 
:$$\varphi _y ( \tau) = \varphi _{xy} ( \tau) * h(- \tau)\quad  \Rightarrow \quad H^{\star}( f ) = \frac{{{\it \Phi} _y ( f )}}{{{\it \Phi} _{xy} ( f )}}.$$
 
:$$\varphi _y ( \tau) = \varphi _{xy} ( \tau) * h(- \tau)\quad  \Rightarrow \quad H^{\star}( f ) = \frac{{{\it \Phi} _y ( f )}}{{{\it \Phi} _{xy} ( f )}}.$$
 +
*In contrast,&nbsp; the first statement is false: &nbsp; The phase relations are lost in the ACF calculation.
 +
*The associated spectral functions to&nbsp; $\varphi_x(\tau)$&nbsp; and&nbsp; $\varphi_x(\tau)$&nbsp; &ndash; namely&nbsp; ${\it \Phi}_x(f)$&nbsp; and&nbsp;  ${\it \Phi}_y(f)$&nbsp; &ndash; are purely real,&nbsp; so only the magnitude&nbsp; $|H(f)|$&nbsp; can be given.
 +
 +
 +
 +
'''(2)'''&nbsp; For Dirac-shaped input ACF&nbsp; $\varphi_x(\tau)$,&nbsp; the impulse response&nbsp; $h(t)$&nbsp; is equal in shape to the CCF:
 +
:$$h(t) = \frac{{K \cdot {\rm{e}}^{ - t/T_0 } }}{N_0 /2} = 1.256 \cdot 10^{ - 2} \frac{1}{{\rm{s}}} \cdot {\rm{e}}^{ - t/T_0 } \hspace{0.3cm}\Rightarrow \hspace{0.3cm}h(t = T_0)\hspace{0.15cm}\underline{ = 4.62 \cdot 10^{-3}\ \rm 1/s}.$$
  
:<b>2.</b>&nbsp;&nbsp;Bei diracförmiger Eingangs-AKF <i>&phi;<sub>x</sub></i>(<i>&tau;</i>) ist die Impulsantwort <i>h</i>(<i>t</i>) formgleich mit der KKF:
 
:$$h(t) = \frac{{K \cdot {\rm{e}}^{ - t/T_0 } }}{N_0 /2} = 1.256 \cdot 10^{ - 2} \frac{1}{{\rm{s}}} \cdot {\rm{e}}^{ - t/T_0 } .$$
 
  
:Für <i>t</i> = <i>T</i><sub>0</sub> ergibt sich der Wert <u>4.62 &middot; 10<sup>&ndash;3</sup> 1/s</u>.
 
  
:<b>3.</b>&nbsp;&nbsp;Die angegebene Fourierkorrespondenz lautet mit <i>T</i><sub>0</sub> = 1/<i>&omega;</i><sub>0</sub> und <i>C</i> = <i>N</i><sub>0</sub>/2 &middot; <i>T</i><sub>0</sub>/<i>K</i>:
+
'''(3)'''&nbsp; The Fourier correspondence given is with&nbsp; $T_0 = 1/\omega_0$&nbsp; and the constant&nbsp; $C= N_0/2 \cdot T_0/K$:
:$$h(t) = \frac{C}{T_0 } \cdot {\rm{e}}^{ - t/T_0 }\circ\!\!-\!\!\!-\!\!\!-\!\!\bullet\, H( \omega  ) = \frac{C}{{1 + {\rm{j}}\omega T_0 }}.$$
+
:$$h(t) = \frac{C}{T_0 } \cdot {\rm{e}}^{ - t/T_0 }\hspace{0.15cm}\circ\!\!-\!\!\!-\!\!\!-\!\!\bullet\,\hspace{0.15cm} H( \omega  ) = \frac{C}{{1 + {\rm{j}}\cdot \omega T_0 }}.$$
  
:Die Konstante ergibt sich zu <i>C</i> = 0.08. Mit <i>H</i>(<i>f</i>) = 2&pi; &middot; <i>H</i>(<i>&omega;</i>) folgt daraus:
+
*The constant results in&nbsp; $C = 0.08$.&nbsp; With&nbsp; $H(f) = 2 \pi \cdot  H(\omega)$&nbsp; it follows:
:$$H(f) = \frac{0.5}{1 + {\rm{j2\pi }}fT_0 }.$$
+
:$$H(f) = \frac{0.5}{1 + {\rm{j\cdot 2\pi }}\cdot f \cdot T_0 } \hspace{0.3cm}\Rightarrow \hspace{0.3cm} H(f= 0) \hspace{0.15cm}\underline{=0.5}.$$
  
:Damit ergibt sich der Gleichsignalübertragungsfaktor <u>zu 0.5</u>.
 
  
:<b>4.</b>&nbsp;&nbsp;Für das Ausgangs-LDS gilt im Allgemeinen bzw. speziell hier:
 
:$${\it \Phi}_y (f) = {\it \Phi} _x (f) \cdot \left| {H(f)} \right|^2  = \frac{N_0 }{2} \cdot \frac{0.5^2 }{{\left( {1 + {\rm{j2\pi }}fT_0 } \right)\left( {1 - {\rm{j2\pi }}fT_0 } \right)}}.$$
 
  
:Dies führt zum Ergebnis:
+
'''(4)'''&nbsp; For the output PSD,&nbsp; in general or specifically here:
:$${\it \Phi}_y (f) = {N_0 }/{8} \cdot \frac{1}{1 + \left( {{\rm{2\pi }}fT_0 } \right)^2 }.$$
+
:$${\it \Phi}_y (f) = {\it \Phi} _x (f) \cdot \left| {H(f)} \right|^2  = \frac{N_0 }{2} \cdot \frac{0.5^2 }{{\left( {1 + {\rm{j\cdot  2\pi }}\cdot f \cdot T_0 } \right)\left( {1 - {\rm{j\cdot 2\pi }}\cdot f \cdot T_0 } \right)}} = {N_0 }/{8} \cdot \frac{1}{1 + \left( {{\rm{2\pi }}\cdot f\cdot T_0 } \right)^2 }.$$
  
:Bei der angegebenen Frequenz ist <i>&Phi;<sub>y</sub></i>(<i>f</i>) gegenüber seinem Maximum um die Hälfte abgefallen:
+
*At the given frequency&nbsp; $f = 1/(2\pi T_0)$,&nbsp; &nbsp; ${\it \Phi}_y (f)$&nbsp; has dropped by half compared to its maximum at $f=0$&nbsp;:
 
:$${\it \Phi}_y (f = 1/(2 \pi T_0)) ={N_0 }/{16}\hspace{0.15cm} \underline{ = 6.25 \cdot 10^{ - 12} \;{\rm{W/Hz}}}.$$
 
:$${\it \Phi}_y (f = 1/(2 \pi T_0)) ={N_0 }/{16}\hspace{0.15cm} \underline{ = 6.25 \cdot 10^{ - 12} \;{\rm{W/Hz}}}.$$
  
Line 88: Line 86:
  
  
[[Category:Aufgaben zu Stochastische Signaltheorie|^5.1 Stochastische Systemtheorie^]]
+
[[Category:Theory of Stochastic Signals: Exercises|^5.1 Stochastic Systems Theory^]]

Latest revision as of 17:54, 22 February 2022

Measuring the frequency response  $H(f)$

We consider the illustrated measurement setup for the determination of the frequency response  $H(f)$  highlighted in blue.

  • The input signal  $x(t)$  is white Gaussian noise with the noise power density  $N_0 = 10^{-10} \hspace{0.05cm} \rm W/Hz$.
  • Thus,  the auto-correlation function  $\rm (ACF)$  is:
$$\varphi _x ( \tau ) = {N_0 }/{2} \cdot \delta ( \tau ).$$
  • The measured cross-correlation function  $\rm (CCF)$  between the signals  $x(t)$  and  $y(t)$  can be approximated as follows
    $($valid only for positive times  $t)$:
$$\varphi _{xy} \left( \tau \right) = K \cdot {\rm{e}}^{ - \tau /T_0 },\hspace{0.5cm}\text{with } \ K = 0.628 \cdot 10^{-12} \hspace{0.05cm} {\rm W}, \ T_0 = 1 \hspace{0.05cm} \rm ms.$$
  • The ACF  $\varphi_y(\tau)$  of the output signal  $y(t)$ is also measured.



Notes:

$$H( \omega ) = \frac{1}{{1 + {\rm{j}}\cdot \omega /\omega _0 }}\hspace{0.15cm}\bullet\!\!\!-\!\!\!-\!\!\!-\!\!\circ\,\hspace{0.15cm}h(t) = \omega _0 \cdot {\rm{e}}^{ - \omega _0 t} \hspace{0.3cm}(t \ge 0).$$
For negative  $t$–values,  on the other hand,  $h(t) =0$  at all times.


Questions

1

Which of the following statements is true?  One can completely determine the frequency response  $H(f)$  by magnitude and phase if

the functions  $\varphi_x(\tau)$  and  $\varphi_y(\tau)$  are known,
the functions  $\varphi_x(\tau)$  and  $\varphi_{xy}(\tau)$  are known,
the functions  $\varphi_{xy}(\tau)$  and  $\varphi_y(\tau)$  are known.

2

Calculate the impulse response  $h(t)$.  What is the value for  $t=T_0$?

$h(t = T_0) \ = \ $

$\ \cdot 10^{-3} \ \rm 1/s$

3

What is the frequency response  $H(f)$?  What value results for $f= 0$?

$H(f = 0) \ = \ $

4

Calculate the power-spectral density of the output signal  $y(t)$.  What value results for frequency  $f = 1/(2\pi T_0)$?

${\it \Phi}_y(f = 1/(2\pi T_0)) \ = \ $

$\ \cdot 10^{-12}\ \rm W/Hz$


Solution

(1)  Statements 2 and 3 are true.

  • The following equations are valid:
$$\varphi _{xy} ( \tau ) = h( \tau ) * \varphi _x ( \tau )\quad \Rightarrow \quad H( f ) = \frac{{{\it \Phi} _{xy} ( f )}}{{{\it \Phi} _x ( f )}},$$
$$\varphi _y ( \tau) = \varphi _{xy} ( \tau) * h(- \tau)\quad \Rightarrow \quad H^{\star}( f ) = \frac{{{\it \Phi} _y ( f )}}{{{\it \Phi} _{xy} ( f )}}.$$
  • In contrast,  the first statement is false:   The phase relations are lost in the ACF calculation.
  • The associated spectral functions to  $\varphi_x(\tau)$  and  $\varphi_x(\tau)$  – namely  ${\it \Phi}_x(f)$  and  ${\it \Phi}_y(f)$  – are purely real,  so only the magnitude  $|H(f)|$  can be given.


(2)  For Dirac-shaped input ACF  $\varphi_x(\tau)$,  the impulse response  $h(t)$  is equal in shape to the CCF:

$$h(t) = \frac{{K \cdot {\rm{e}}^{ - t/T_0 } }}{N_0 /2} = 1.256 \cdot 10^{ - 2} \frac{1}{{\rm{s}}} \cdot {\rm{e}}^{ - t/T_0 } \hspace{0.3cm}\Rightarrow \hspace{0.3cm}h(t = T_0)\hspace{0.15cm}\underline{ = 4.62 \cdot 10^{-3}\ \rm 1/s}.$$


(3)  The Fourier correspondence given is with  $T_0 = 1/\omega_0$  and the constant  $C= N_0/2 \cdot T_0/K$:

$$h(t) = \frac{C}{T_0 } \cdot {\rm{e}}^{ - t/T_0 }\hspace{0.15cm}\circ\!\!-\!\!\!-\!\!\!-\!\!\bullet\,\hspace{0.15cm} H( \omega ) = \frac{C}{{1 + {\rm{j}}\cdot \omega T_0 }}.$$
  • The constant results in  $C = 0.08$.  With  $H(f) = 2 \pi \cdot H(\omega)$  it follows:
$$H(f) = \frac{0.5}{1 + {\rm{j\cdot 2\pi }}\cdot f \cdot T_0 } \hspace{0.3cm}\Rightarrow \hspace{0.3cm} H(f= 0) \hspace{0.15cm}\underline{=0.5}.$$


(4)  For the output PSD,  in general or specifically here:

$${\it \Phi}_y (f) = {\it \Phi} _x (f) \cdot \left| {H(f)} \right|^2 = \frac{N_0 }{2} \cdot \frac{0.5^2 }{{\left( {1 + {\rm{j\cdot 2\pi }}\cdot f \cdot T_0 } \right)\left( {1 - {\rm{j\cdot 2\pi }}\cdot f \cdot T_0 } \right)}} = {N_0 }/{8} \cdot \frac{1}{1 + \left( {{\rm{2\pi }}\cdot f\cdot T_0 } \right)^2 }.$$
  • At the given frequency  $f = 1/(2\pi T_0)$,    ${\it \Phi}_y (f)$  has dropped by half compared to its maximum at $f=0$ :
$${\it \Phi}_y (f = 1/(2 \pi T_0)) ={N_0 }/{16}\hspace{0.15cm} \underline{ = 6.25 \cdot 10^{ - 12} \;{\rm{W/Hz}}}.$$